Difference between revisions of "1975 AHSME Problems/Problem 16"

(Created page with "==Problem== If the first term of an infinite geometric series is a positive integer, the common ratio is the reciprocal of a positive integer, and the sum of the series is <ma...")
 
 
Line 9: Line 9:
 
\textbf{(E)}\ \frac{9}{2} \qquad
 
\textbf{(E)}\ \frac{9}{2} \qquad
 
</math>
 
</math>
 +
 +
==Solution==
 +
nothing yet :(
 +
 +
==See Also==
 +
{{AHSME box|year=1975|num-b=15|num-a=17}}
 +
{{MAA Notice}}

Latest revision as of 17:17, 19 January 2021

Problem

If the first term of an infinite geometric series is a positive integer, the common ratio is the reciprocal of a positive integer, and the sum of the series is $3$, then the sum of the first two terms of the series is

$\textbf{(A)}\ \frac{1}{3} \qquad \textbf{(B)}\ \frac{2}{3} \qquad \textbf{(C)}\ \frac{8}{3} \qquad \textbf{(D)}\ 2           \qquad \textbf{(E)}\ \frac{9}{2} \qquad$

Solution

nothing yet :(

See Also

1975 AHSME (ProblemsAnswer KeyResources)
Preceded by
Problem 15
Followed by
Problem 17
1 2 3 4 5 6 7 8 9 10 11 12 13 14 15 16 17 18 19 20 21 22 23 24 25 26 27 28 29 30
All AHSME Problems and Solutions

The problems on this page are copyrighted by the Mathematical Association of America's American Mathematics Competitions. AMC logo.png